4
$\begingroup$

Let $f$ be an entire function such that : $ |f(z)| \leqslant 1 + |z|^{\frac{3} {2}} \forall z $

What we can conclude about $f$ . Sorry for asking this , but I want to see some examples of the contents of the chapter that I'm reading, this problem it's from the chapter of maximum modulus principle.

  • 0
    Ok here is another one question with general solution $f$or this kind o$f$ problems http://math.stackexchange.com/questions/171610/an-entire-function-is-identically-zero/1716152012-10-07

1 Answers 1

1

Edit: oop... misread... revised: The given inequality and Cauchy's formula for the second derivative $f''$, letting the large circle go to infinity, show that $f''(z)=0$, so $f$ is (not constant, but) linear. This is just a little extension of the argument for Liouville's theorem, so not really so much about maximum modulus, perhaps.

Edit-edit: explicitly, by the Cauchy integral formula for the derivatives, $f''(z)={2!\over 2\pi i}\int_\gamma {f(\zeta)\,d\zeta\over (\zeta-z)^3}$, where $\gamma$ is a large circle of radius $R$. The numerator is bounded by $R^{3/2}$, and the denominator is essentially $R^3$. The length of the curve is $2\pi R$, so the integral expressing the second derivative is bounded by a constant multiple of $1/R^{1/2}$, which goes to $0$ as $R$ goes to $+\infty$.

  • 0
    @paulgarrett could you write the estimates and inequality for me please? I am having problem to get $f''(0)=0$2013-05-07